LSAT and Law School Admissions Forum

Get expert LSAT preparation and law school admissions advice from PowerScore Test Preparation.

 est15
  • Posts: 94
  • Joined: Aug 28, 2013
|
#15811
I can see reasons for both B and E being correct. I thought B worked because having to pay the scientists more and more each year explains why even though funding has increased over the past ten years, the current amount is still not enough. Why doesn't B help resolve the paradox?
 Nikki Siclunov
PowerScore Staff
  • PowerScore Staff
  • Posts: 1362
  • Joined: Aug 02, 2011
|
#15822
Hi est15,

The stimulus describes how the amount of funding for wetlands preservation has tripled over the last 10 years, and yet the actual amount is still not enough and should be increased. There is no paradox here. Since the initial amount of funding is never stated, it is entirely possible that tripling a really low amount is still not enough. Answer choice (E) nails the issue on the head, and thus strengthens the scientist's conclusion.

From answer choice (B) we know that the salaries of scientists have increased at a rate higher than the rate of inflation. So what? First, we don't know the rate of inflation, and second - we have no idea what proportion of the overall budget is allocated to cover scientists' salaries. What if the rate of inflation is 3%, and their salaries increased by 4%? What if their salaries constitute only 1% of the government funding for wetlands preservation? Either way, just because you have a marginal increase in one type of (potentially insignificant) expense does not explain why tripling the budget will prove inadequate. That's like complaining that a 3-fold increase in your salary isn't enough because your rent increased faster than inflation.

This is a great example of a question where you absolutely had to focus on the numbers, notice the missing piece of information from the stimulus (what was the budget 10 years ago?) and prephrase the correct answer choice.

Hope this helps!

Let me know.
 LSAT2018
  • Posts: 242
  • Joined: Jan 10, 2018
|
#48091
When you say 'a sixfold increase in government funding for the preservation of wetlands' and 'the total area of wetlands needing such preservation has increased only twofold' does sixfold increase and twofold indicate a percentage here? So in the stimulus a percentage is compared with another percentage.
 HowardQ
  • Posts: 32
  • Joined: Jun 25, 2018
|
#48857
Hi,
I still don't see how B is a better answer than D. At least B stated that the cost increased, while E did not mention even an increase in the budget associated with parks. It is also possible that the park still needs more funding because there still need more scientist, which due to the high salary they were unable to obtain. A criticism of the rate of growth of salaries, but E seems to provide an even broader range of uncertainty since on a number scale almost nonexistent means an infinitely small number which provides more uncertainty to the 3-time difference idea. The maximum discrepancy B have is 3 times, but the maximum discrepancy E has is unlimited. For example, a change from almost none $1 to $100000 can have a much larger difference. Could anyone help me find an explanation?


Thanks,

Howard
User avatar
 Dave Killoran
PowerScore Staff
  • PowerScore Staff
  • Posts: 5852
  • Joined: Mar 25, 2011
|
#49170
LSAT2018 wrote:When you say 'a sixfold increase in government funding for the preservation of wetlands' and 'the total area of wetlands needing such preservation has increased only twofold' does sixfold increase and twofold indicate a percentage here? So in the stimulus a percentage is compared with another percentage.
Hi LSAT,

Yes, it does work out that way. The references to "twofold" and "sixfold" are straight multiples. Twofold means twice as much, and sixfold means six times as much. Now, those can also be expressed as basic percentages (200% and 600%), so the functionality is the same. Does that make sense?

Thanks!
User avatar
 Dave Killoran
PowerScore Staff
  • PowerScore Staff
  • Posts: 5852
  • Joined: Mar 25, 2011
|
#49171
HowardQ wrote:Hi,
I still don't see how B is a better answer than D. At least B stated that the cost increased, while E did not mention even an increase in the budget associated with parks. It is also possible that the park still needs more funding because there still need more scientist, which due to the high salary they were unable to obtain. A criticism of the rate of growth of salaries, but E seems to provide an even broader range of uncertainty since on a number scale almost nonexistent means an infinitely small number which provides more uncertainty to the 3-time difference idea. The maximum discrepancy B have is 3 times, but the maximum discrepancy E has is unlimited. For example, a change from almost none $1 to $100000 can have a much larger difference. Could anyone help me find an explanation?


Thanks,

Howard
Hi Howard,

It seems you've posted a question for a different problem than the one here. I can't seem to identify it quickly here—is it for this test?

Thanks!
 HowardQ
  • Posts: 32
  • Joined: Jun 25, 2018
|
#49182
Dave Killoran wrote:
Hi Howard,

It seems you've posted a question for a different problem than the one here. I can't seem to identify it quickly here—is it for this test?

Thanks!
Hi Dave,
Sorry, I don't know how the answer choices, let me rephrase this
Hi,
I see B as a better answer choice than E. Both these answers provide a reason for the described 6 fold versus 2 folds, but B provides less range of possible answers. First of all, B provides an answer that cost of wetland preservation for sure increased faster 2 folds, which indicates that 6 fold is possible. On the other hand, choice E indicated that anything is possible since an original value close to 0 indicates the method of counting folds is flawed. Overall I think B has more probability to arrive at the statistic described rather than E.


Thanks,

Howard
User avatar
 Dave Killoran
PowerScore Staff
  • PowerScore Staff
  • Posts: 5852
  • Joined: Mar 25, 2011
|
#49188
Hi Howard,

Ok, great, now I see the direction you were going in. Your original message talked about B being better than D, and that was confusing for me :-D

Let's talk about this one a bit more, because it's a great example of the type of problem you want to make sure you understand going forward. First, answer choice (E) is definitely the better answer, but the key here isn't what I think, it's what the test makers think. Even though you think (B) is better, the way to analyze LSAT problems is to put aside your interpretation and try to understand their interpretation. They think (E) is better, so try to understand their mindset as well as possible.

Second, in his response above, Nikki lays out a convincing case for why (B) is irrelevant so I won't repeat that but I recommend you read what he said. In short, the salaries are a just a single component of the overall budget, and all we've been told about is the overall funding. The movement of individual components inside that larger figure is going to have to do a lot more work than (B) does to resolve the stimulus issue.

Last, with (E), let's try an analogy in the form of an exchange between two people and see if we can make it clear why it works. I'll drop the inflation issue which is largely just a distractor element:

  • Parent: In the last year, we've increased your allowance six times while your spending needs have only doubled.

    Child: Nonetheless, my allowance is still woefully inadequate and should be increased.

    Question: Which one of the following would allow both Parent and Child to be correct and explain the need for a greater allowance?

    Answer choice (E): A year ago, the child's allowance was just a few cents.

The reason (E) works above is that it shows the starting point for funding/allowance matters. If the initial funding was very small, increasing it by 6 times is still a relatively small number. Essentially, if the initial amount was almost nonexistent and far too small to cover the needs then, you'd probably need to increase it many times more to make it match the needs now, especially since the land area has doubled. Since this angle allows for an explanation of the above, this is the correct answer.

This is a classic numbers and percentages problem, so make sure to study this one closely.

Thanks!
 ericj_williams
  • Posts: 63
  • Joined: Jan 19, 2020
|
#85179
Dave Killoran wrote: Sat Aug 04, 2018 11:47 am Hi Howard,

Ok, great, now I see the direction you were going in. Your original message talked about B being better than D, and that was confusing for me :-D

Let's talk about this one a bit more, because it's a great example of the type of problem you want to make sure you understand going forward. First, answer choice (E) is definitely the better answer, but the key here isn't what I think, it's what the test makers think. Even though you think (B) is better, the way to analyze LSAT problems is to put aside your interpretation and try to understand their interpretation. They think (E) is better, so try to understand their mindset as well as possible.

Second, in his response above, Nikki lays out a convincing case for why (B) is irrelevant so I won't repeat that but I recommend you read what he said. In short, the salaries are a just a single component of the overall budget, and all we've been told about is the overall funding. The movement of individual components inside that larger figure is going to have to do a lot more work than (B) does to resolve the stimulus issue.

Last, with (E), let's try an analogy in the form of an exchange between two people and see if we can make it clear why it works. I'll drop the inflation issue which is largely just a distractor element:

  • Parent: In the last year, we've increased your allowance six times while your spending needs have only doubled.

    Child: Nonetheless, my allowance is still woefully inadequate and should be increased.

    Question: Which one of the following would allow both Parent and Child to be correct and explain the need for a greater allowance?

    Answer choice (E): A year ago, the child's allowance was just a few cents.

The reason (E) works above is that it shows the starting point for funding/allowance matters. If the initial funding was very small, increasing it by 6 times is still a relatively small number. Essentially, if the initial amount was almost nonexistent and far too small to cover the needs then, you'd probably need to increase it many times more to make it match the needs now, especially since the land area has doubled. Since this angle allows for an explanation of the above, this is the correct answer.

This is a classic numbers and percentages problem, so make sure to study this one closely.

Thanks!
What's wrong with C. If we can identify not only wetlands that needs preservation now, but will need it in the future, wouldn't that justify the increase in spending, as we are spending money on wetlands that definitely need preservation now as well as in the future?
 Rachael Wilkenfeld
PowerScore Staff
  • PowerScore Staff
  • Posts: 1358
  • Joined: Dec 15, 2011
|
#85294
Answer choice (C) is a great example of an answer that doesn't really address the facts as we have them, Eric. In our stimulus, it tells us that the wetlands in need of protection have only increased twofold. Even if answer choice (C) is true, that wouldn't change the increase in wetlands needing protection. They are already considered as part of the argument. It doesn't matter why the increase occurred. Maybe, as in answer choice (C), scientists got better at identifying wetlands that needed protection. Maybe there was another reason, like human activity began to threaten more wetlands. The reason for the increase does not change the statistics of the increase.

What we are looking for here is a reason that the wetlands needing protection increased only two fold, funding increased three times the prior level, but we still don't have enough. Why wasn't the triple-funding enough? What are the numbers not telling us?

We might make an assumption here. Maybe it's an assumption you made. Either way, it would be a mistake. We can't assume that the amount of funding 10 years ago was anywhere near adequate. That's the key here. If the funding was inadequate 10 years ago, especially if it was very inadequate, even a sixfold increase may not be enough. We have to know the starting number, not just the change in percentages.

Hope that helps!

Get the most out of your LSAT Prep Plus subscription.

Analyze and track your performance with our Testing and Analytics Package.